What is wrong with this solution? \(\displaystyle{\ln{{\left({x}\right)}}}={1}+{\ln{{\left({5}\right)}}}\) \(\displaystyle{x}={e}^{{{1}+{\ln{{\left({5}\right)}}}}}={e}^{{{1}+{5}}}={e}^{{6}}\) What

Charity Odom

Charity Odom

Answered question

2022-03-12

What is wrong with this solution?
ln(x)=1+ln(5)
x=e1+ln(5)=e1+5=e6
What exactly am I doing wrong here?

Answer & Explanation

elementalfoxwqe

elementalfoxwqe

Beginner2022-03-13Added 3 answers

You change ln(5) to 5 in the exponent, when it should be
e1+ln(5)=eeln(5)=5e.
Pentyrch9ci

Pentyrch9ci

Beginner2022-03-14Added 9 answers

lnx=1+ln5
lnx=lne+ln5
lnx=ln(5e)
x=5e

Do you have a similar question?

Recalculate according to your conditions!

Ask your question.
Get an expert answer.

Let our experts help you. Answer in as fast as 15 minutes.

Didn't find what you were looking for?